Difference between revisions of "2021 JMPSC Invitationals Problems/Problem 9"
(→Solution) |
(→Solution) |
||
Line 4: | Line 4: | ||
==Solution== | ==Solution== | ||
From the fact that <math>AD=DB</math> and <math>\angle ADC = 2\angle ABC,</math> we find that <math>\triangle ABC</math> is a right triangle with a right angle at <math>C;</math> thus by the Pythagorean Theorem we obtain <math>AC=\boxed{24}.</math> | From the fact that <math>AD=DB</math> and <math>\angle ADC = 2\angle ABC,</math> we find that <math>\triangle ABC</math> is a right triangle with a right angle at <math>C;</math> thus by the Pythagorean Theorem we obtain <math>AC=\boxed{24}.</math> | ||
+ | |||
+ | ==Solution== | ||
+ | Note that <math>\angle BDC = 180-x</math>, which means <math>\angle DCB = \angle DBC</math> and <math>AD=DB=DC=13</math>. Now, Stewart's Theorem dictates <math>x^2 \cdot 13 = 7488</math>, yielding <math>AC=x=\boxed{24}</math> ~Geometry285 | ||
==See also== | ==See also== |
Revision as of 17:10, 11 July 2021
Contents
Problem
In , let be on such that . If , , and , find
Solution
From the fact that and we find that is a right triangle with a right angle at thus by the Pythagorean Theorem we obtain
Solution
Note that , which means and . Now, Stewart's Theorem dictates , yielding ~Geometry285
See also
- Other 2021 JMPSC Invitationals Problems
- 2021 JMPSC Invitationals Answer Key
- All JMPSC Problems and Solutions
The problems on this page are copyrighted by the Junior Mathematicians' Problem Solving Competition.